AP Government: Landmark Supreme Court Cases long version

Pataasin ang iyong marka sa homework at exams ngayon gamit ang Quizwiz!

Alden v. Main (1999)

Court held that state employees could not sue to force state compliance with federal-fair labor laws

McConnell v. Federal Election Commission (2003)

Facts of the Case In early 2002, a many years-long effort by Senators John McCain and Russell Feingold to reform the way that money is raised for--and spent during--political campaigns culminated in the passage of the Bipartisan Campaign Finance Reform Act of 2002 (the so-called McCain-Feingold bill). Its key provisions were a) a ban on unrestricted ("soft money") donations made directly to political parties (often by corporations, unions, or well-healed individuals) and on the solicitation of those donations by elected officials; b) limits on the advertising that unions, corporations, and non-profit organizations can engage in up to 60 days prior to an election; and c) restrictions on political parties' use of their funds for advertising on behalf of candidates (in the form of "issue ads" or "coordinated expenditures"). The campaign finance reform bill contained an unusual provision providing for an early federal trial and a direct appeal to the Supreme Court of the United States, by-passing the typical federal judicial process. In May a special three-judge panel struck down portions of the Campaign Finance Reform Act's ban on soft-money donations but upheld some of the Act's restrictions on the kind of advertising that parties can engage in. The ruling was stayed until the Supreme Court could hear and decide the resulting appeals. Question Presented 1. Does the "soft money" ban of the Campaign Finance Reform Act of 2002 exceed Congress's authority to regulate elections under Article 1, Section 4 of the United States Constitution and/or violate the First Amendment's protection of the freedom to speak? 2. Do regulations of the source, content, or timing of political advertising in the Campaign Finance Reform Act of 2002 violate the First Amendment's free speech clause? Conclusion With a few exceptions, the Court answered "no" to both questions in a 5-to-4 decision written by Justices Sandra Day O'Connor and John Paul Stevens. Because the regulations dealt mostly with soft-money contributions that were used to register voters and increase attendance at the polls, not with campaign expenditures (which are more explicitly a statement of political values and therefore deserve more protection), the Court held that the restriction on free speech was minimal. It then found that the restriction was justified by the government's legitimate interest in preventing "both the actual corruption threatened by large financial contributions and... the appearance of corruption" that might result from those contributions. In response to challenges that the law was too broad and unnecessarily regulated conduct that had not been shown to cause corruption (such as advertisements paid for by corporations or unions), the Court found that such regulation was necessary to prevent the groups from circumventing the law. Justices O'Connor and Stevens wrote that "money, like water, will always find an outlet" and that the government was therefore justified in taking steps to prevent schemes developed to get around the contribution limits. The Court also rejected the argument that Congress had exceeded its authority to regulate elections under Article I, Section 4 of the Constitution. The Court found that the law only affected state elections in which federal candidates were involved and also that it did not prevent states from creating separate election laws for state and local elections.

*NAACP v. Alabama (1958)

*NAACP v. Alabama (1958) Subjects: Judicial Power: Standing to Sue, Personal Injury Facts of the Case As part of its strategy to enjoin the NAACP from operating, Alabama required it to reveal to the State's Attorney General the names and addresses of all the NAACP's members and agents in the state. Question Presented Did Alabama's requirement violate the Due Process Clause of the Fourteenth Amendment? Conclusion Yes. The unanimous Court held that a compelled disclosure of the NAACP's membership lists would have the effect of suppressing legal association among the group's members. Nothing short of an "overriding valid interest of the State," something not present in this case, was needed to justify Alabama's actions.

*Brown v. Board of Education II (1955)

*Brown v. Board of Education II (1955) Subjects: Civil Rights: Desegregation, Schools Facts of the Case After its decision in Brown I which declared racial discrimination in public education unconstitutional, the Court convened to issue the directives which would help to implement its newly announced Constitutional principle. Given the embedded nature of racial discrimination in public schools and the diverse circumstances under which it had been practiced, the Court requested further argument on the issue of relief. Question Presented What means should be used to implement the principles announced in Brown I? Conclusion The Court held that the problems identified in Brown I required varied local solutions. Chief Justice Warren conferred much responsibility on local school authorities and the courts which originally heard school segregation cases. They were to implement the principles which the Supreme Court embraced in its first Brown decision. Warren urged localities to act on the new principles promptly and to move toward full compliance with them "with all deliberate speed."

Thompson v. Oklahoma (1988)

At the age of 15, William Thompson brutally murdered his brother-in-law, who had been abusing his sister. According to Oklahoma law, Thompson was a minor and must be tried as one, thereby receiving a reduced sentence and other considerations. Due to the gravity of the crime, the prosecutor requested an order which would allow the boy to be tried as an adult. In order to do this, the prosecution had to demonstrate that the case had merit and that the chance for rehabilitation of the child within the juvenile system was slim. An Oklahoma District court granted the order after hearing of the boy's violent nature, past offenses, and psychiatric testimony. Thompson was convicted of first-degree murder and sentenced to death. The ruling was appealed and the Supreme Court overturned the death sentence, holding that "[t]he Eighth and Fourteenth Amendments prohibit the execution of a person who was under 16 years of age at the time of his or her offense." The Court thereby ruled that applying the death penalty to a 15 year old was prohibited under the Constitution's cruel and unusual punishment clause

Gitlow v. New York (1925)

Benjamin Gitlow had been a prominent member of the Socialist party during the 1920s. He was arrested and convicted for violating the New York Criminal Anarchy Law of 1902, which made it a crime to attempt to foster the violent overthrow of government. Gitlow's publication and circulation of sixteen thousand copies of the Left-Wing Manifesto violated this Criminal Anarchy Act. The pamphlet went on to advocate the creation of a socialist system through the use of massive strikes and "class action...in any form." Gitlow was tried and convicted. He appealed the decision, arguing that his First Amendment right to freedoms of speech and press was violated. Although the New York courts held that the Communists must be held accountable for the results of their propaganda, the Supreme Court ruled in favor of Gitlow. It stated in its decision that "for present purposes, we may assume that freedom of speech and of press...are among the fundamental personal rights and liberties protected by the due process clause of the Fourteenth Amendment from impairment by the State."

Betts v. Brady (1942)

Betts was indicted for robbery and detained in a Maryland jail. Prior to his trial, he asked for counsel to represent him. This request was denied and he was soon convicted. While incarcerated, Betts filed a habeas corpus petition in the lower courts. After they rejected his petitions, he filed a certiorari petition with the Supreme Court, which agreed to hear his case. Bett argued that his 6th Amendment right to a fair trial was violated because of his lack of counsel. The State of Maryland held that most states did not require the appointment of counsel in non-capital cases and the circumstances of this particular case did not require it. Although the Court found in favor of Betts, it decided that the right to counsel must be decided on a case- by-case basis. This ruling was upheld for 20 years until it was overturned by Gideon v. Wainwright in 1963.

Chimel v. California (6-3 vote, June 23, 1969)

Chimel concerns the right of the police to search in the vicinity of an arrest. It involved a man who had been burglarizing a specific area for quite some time. The police knew this and swore out a warrant for his arrest. Upon arresting him, they searched his apartment and discovered evidence that implicated him in the crimes. The search, however was done without a warrant. The police argued that they had merely followed the law which allowed them to search within the vicinity of an arrest in order to prevent escape, the destruction of evidence, or the acquisition of a weapon. The Court upheld the search, arguing "the area in which an arrestee might reach in order to grab a weapon or evidentiary items must, of course, be governed by a like rule. A gun on a table or in a drawer in front of one who is arrested can be as dangerous to the arresting officer as one concealed in the clothing of the person arrested. There is ample justification, therefore, for a search of the arrestee's person and the area 'within his immediate control.'"

Gideon v. Wainwright (1963)

Clarence Earl Gideon was arrested for attempting to break and enter a poolroom with intent to commit a misdemeanor. Under Florida state law, this was a felony. Gideon, who was illiterate and had no money to hire a competent attorney, requested council be appointed. At the time, such appointment of council was only required by law in cases where the defendant was facing capital punishment. The trial judge ruled that Gideon was competent enough to handle his own defense as well as any man. Gideon was convicted and sent to prison, where he filed a habeas corpus petition in the Supreme Court. The Court agreed to hear his case and council was appointed to him. The Supreme Court found in favor of Gideon, overruling Betts v. Brady. In doing so, it declared that all defendents in felony cases had the right to an attorney. Justice Hugo Black, in writing for the majority, stated that the Court was "returning to...old precedents, sounder we believe than the new." Gideon was given a new trial with a lawyer and the jury acquitted him.

Ex parte Milligan (1866)

During the Civil War, with civil liberties in the North being constricted, officials of the United States arrested several antiwar Democrats in Indiana. The president feared the weak support for the war in Indiana would lead to an acquittal by an Indiana jury. Therefore, the politicians were not given a trial by jury, but rather were held as military prisoners and convicted. One of the defendents, Milligan, appealed. The Supreme Court found in favor of the defendants, stating "the Constitution...is a law for the rulers and people, equally in war and peace, and covers...all...men, at all times, and under all circumstances." The Court went on to hold that the president's power to suspend the writ of habeas corpus in a time of war did not extend to creating another court system run by the military.

Griffin v. California (1965)

Eddie Dean Griffin had been accused of assaulting and murdering a female friend of his. To these charges, he plead not guilty. Griffin's counsel recommended that he not testify on the grounds that the prosecution's case was entirely circumstantial. During the trial, the prosecutor brought into evidence the fact that Griffin had been seen in the alley where the victim was found and had left it "cool as a cucumber". The main thrust of his closing statement, however, concerned the defendant's refusal to testify. The jury found Griffin guilty and sentenced him to death. Griffin appealed the case on the grounds that he was denied his Fifth Amendment right refrain from testifying as a defense. The Supreme Court found in his favor. They reasoned that by referring to the defendant's lack of testimony in front of the jury, the prosecution denied him his Fifth and Fourteenth Amendment rights. The Court went on to say what the jury "may infer when the court solemnizes the silence of the accused into evidence against him is [dangerous]."

Escobedo v. Illinois (1964)

Escobedo was arrested in connection with a murder and brought to the police station. He repeatedly asked to see his lawyer, but was never allowed out of the interrogation room. His lawyer even went so far as to come to the police station in search of him, but was denied access. Escobedo then confessed while under interrogation to firing the shot that killed the victim. As a result, he was soon convicted. Escobedo appealed to the Supreme Court and it overturned the conviction. The Court extended the "exclusionary rule" to illegal confessions and ruled that Escobedo's confession should not have been allowed in as evidence. The Court also defined the "Escobedo Rule" which holds that individuals have the right to an attorney when an "investigation is no longer a general inquiry...but has begun to focus on a particular suspect..." The ruling went on to detail that (Where) the suspect has been taken into custody...the suspect has requested...his lawyer, and the police have not...warned him of his right to remain silent, the accused has been denied...counsel in violation of the Sixth Amendment."

*Everson v. Board of Education (1947)

Facts of the Case A New Jersey law allowed reimbursements of money to parents who sent their children to school on buses operated by the public transportation system. Children who attended Catholic schools also qualified for this transportation subsidy. Question Presented Did the New Jersey statute violate the Establishment Clause of the First Amendment as made applicable to the states through the Fourteenth Amendment? Conclusion No. A divided Court held that the law did not violate the Constitution. After detailing the history and importance of the Establishment Clause, Justice Black argued that services like bussing and police and fire protection for parochial schools are "separate and so indisputably marked off from the religious function" that for the state to provide them would not violate the First Amendment. The law did not pay money to parochial schools, nor did it support them directly in anyway. It was simply a law enacted as a "general program" to assist parents of all religions with getting their children to school.

Everson v. Board of Education (1947)

Facts of the Case A New Jersey law allowed reimbursements of money to parents who sent their children to school on buses operated by the public transportation system. Children who attended Catholic schools also qualified for this transportation subsidy. Question Presented Did the New Jersey statute violate the Establishment Clause of the First Amendment as made applicable to the states through the Fourteenth Amendment? Conclusion No. A divided Court held that the law did not violate the Constitution. After detailing the history and importance of the Establishment Clause, Justice Black argued that services like bussing and police and fire protection for parochial schools are "separate and so indisputably marked off from the religious function" that for the state to provide them would not violate the First Amendment. The law did not pay money to parochial schools, nor did it support them directly in anyway. It was simply a law enacted as a "general program" to assist parents of all religions with getting their children to school.

* Gibbons v. Ogden (1824)

Facts of the Case A New York state law gave two individuals the exclusive right to operate steamboats on waters within state jurisdiction. Laws like this one were duplicated elsewhere which led to friction as some states would require foreign (out-of-state) boats to pay substantial fees for navigation privileges. In this case a steamboat owner who did business between New York and New Jersey challenged a law which forced him to obtain an operating permit from the State of New York to navigate on that state's waters. Question Presented Did the State of New York exercise authority in a realm reserved exclusively to Congress, namely, the regulation of interstate commerce? Conclusion The Court found that New York's licensing requirement for out-of-state operators was inconsistent with a congressional act regulating the coasting trade. The New York law was invalid by virtue of the Supremacy Clause. In his opinion, Chief Justice Marshall developed a clear definition of the word commerce, which included navigation on interstate waterways. He also gave meaning to the phrase "among the several states" in the Commerce Clause. Marshall's was one of the earliest and most influential opinions concerning this important clause. He concluded that regulation of navigation by steamboat operators and others for purposes of conducting interstate commerce was a power reserved to and exercised by the Congress.

Smith v. Allwright (1944)

Facts of the Case A resolution of the Democratic Party of Texas, a group that the Texas Supreme Court had deemed a "voluntary association," allowed only whites to participate in Democratic primary elections. S.S. Allwright was a county election official; he denied Lonnie E. Smith, a black man, the right to vote in the 1940 Texas Democratic primary. Question Presented Did denying blacks the right to vote in primary elections violate the Fifteenth Amendment? Conclusion The Court overruled its decision in Grovey v. Townsend (1935) and found the restrictions against blacks unconstitutional. Even though the Democratic Party was a voluntary organization, the fact that Texas statutes governed the selection of county-level party leaders, the party conducted primary elections under state statutory authority, and state courts were given exclusive original jurisdiction over contested elections, guaranteed for blacks the right to vote in primaries. Allwright engaged in state action abridging Smith's right to vote because of his race. A state cannot "permit a private organization to practice racial discrimination" in elections, argued Justice Reed. (The Court's decision in this matter was amended on June 12, 1944.)

Bethel School District No. 403 v. Fraser (1986)

Facts of the Case At a school assembly of approximately 600 high school students, Matthew Fraser made a speech nominating a fellow student for elective office. In his speech, Fraser used what some observers believed was a graphic sexual metaphor to promote the candidacy of his friend. As part of its disciplinary code, Bethel High School enforced a rule prohibiting conduct which "substantially interferes with the educational process . . . including the use of obscene, profane language or gestures." Fraser was suspended from school for two days. Question Presented Does the First Amendment prevent a school district from disciplining a high school student for giving a lewd speech at a high school assembly? Conclusion No. The Court found that it was appropriate for the school to prohibit the use of vulgar and offensive language. Chief Justice Burger distinguished between political speech which the Court previously had protected in Tinker v. Des Moines Independent Community School District (1969) and the supposed sexual content of Fraser's message at the assembly. Burger concluded that the First Amendment did not prohibit schools from prohibiting vulgar and lewd speech since such discourse was inconsistent with the "fundamental values of public school education."

*Dred Scott v. Sandford (1857)

Facts of the Case Dred Scott was a slave in Missouri. From 1833 to 1843, he resided in Illinois (a free state) and in an area of the Louisiana Territory, where slavery was forbidden by the Missouri Compromise of 1820. After returning to Missouri, Scott sued unsuccessfully in the Missouri courts for his freedom, claiming that his residence in free territory made him a free man. Scott then brought a new suit in federal court. Scott's master maintained that no pure-blooded Negro of African descent and the descendant of slaves could be a citizen in the sense of Article III of the Constitution. Question Presented Was Dred Scott free or slave? Conclusion Dred Scott was a slave. Under Articles III and IV, argued Taney, no one but a citizen of the United States could be a citizen of a state, and that only Congress could confer national citizenship. Taney reached the conclusion that no person descended from an American slave had ever been a citizen for Article III purposes. The Court then held the Missouri Compromise unconstitutional, hoping to end the slavery question once and for all.

*Schenck v. United States (1919)

Facts of the Case During World War I, Schenck mailed circulars to draftees. The circulars suggested that the draft was a monstrous wrong motivated by the capitalist system. The circulars urged "Do not submit to intimidation" but advised only peaceful action such as petitioning to repeal the Conscription Act. Schenck was charged with conspiracy to violate the Espionage Act by attempting to cause insubordination in the military and to obstruct recruitment. Question Presented Are Schenck's actions (words, expression) protected by the free speech clause of the First Amendment? Conclusion Holmes, speaking for a unanimous Court, concluded that Schenck is not protected in this situation. The character of every act depends on the circumstances. "The question in every case is whether the words used are used in such circumstances and are of such a nature as to create a clear and present danger that they will bring about the substantive evils that Congress has a right to prevent." During wartime, utterances tolerable in peacetime can be punished.

*Korematsu v. United States (1944)

Facts of the Case During World War II, Presidential Executive Order 9066 and congressional statutes gave the military authority to exclude citizens of Japanese ancestry from areas deemed critical to national defense and potentially vulnerable to espionage. Korematsu remained in San Leandro, California and violated Civilian Exclusion Order No. 34 of the U.S. Army. Question Presented Did the President and Congress go beyond their war powers by implementing exclusion and restricting the rights of Americans of Japanese descent? Conclusion The Court sided with the government and held that the need to protect against espionage outweighed Korematsu's rights. Justice Black argued that compulsory exclusion, though constitutionally suspect, is justified during circumstances of "emergency and peril."

Palko v. Connecticut (1937)

Facts of the Case Frank Palko had been charged with first-degree murder. He was convicted instead of second-degree murder and sentenced to life imprisonment. The state of Connecticut appealed and won a new trial; this time the court found Palko guilty of first-degree murder and sentenced him to death. Question Presented Does Palko's second conviction violate the protection against double jeopardy guaranteed by the Fifth Amendment because this protection applies to the states by virtue of the Fourteenth Amendment's due process clause? Conclusion The Supreme Court upheld Palko's second conviction. In his majority opinion, Cardozo formulated principles that were to direct the Court's actions for the next three decades. He noted that some Bill of Rights guarantees--such as freedom of thought and speech--are fundamental, and that the Fourteenth Amendment's due process clause absorbed these fundamental rights and applied them to the states. Protection against double jeopardy was not a fundamental right. Palko died in Connecticut's gas chamber in April 1938.

*Gitlow v. New York (1925)

Facts of the Case Gitlow, a socialist, was arrested for distributing copies of a "left-wing manifesto" that called for the establishment of socialism through strikes and class action of any form. Gitlow was convicted under a state criminal anarchy law, which punished advocating the overthrow of the government by force. At his trial, Gitlow argued that since there was no resulting action flowing from the manifesto's publication, the statute penalized utterences without propensity to incitement of concrete action. The New York courts had decided that anyone who advocated the doctrine of violent revolution violated the law. Question Presented Does the New York law punishing the advocacy of overthrowing the government an unconstitutional violation of the free speech clause of the First Amendment? Conclusion Threshold issue: Does the First Amendment apply to the states? Yes, by virtue of the liberty protected by due process that no state shall deny (14th Amendment). On the merits, a state may forbid both speech and publication if they have a tendency to result in action dangerous to public security, even though such utterances create no clear and present danger. The rationale of the majority has sometimes been called the "dangerous tendency" test. The legislature may decide that an entire class of speech is so dangerous that it should be prohibited. Those legislative decisions will be upheld if not unreasonable, and the defendant will be punished even if her speech created no danger at all.

*Fletcher v. Peck (1810)

Facts of the Case In 1795, the Georgia state legislature passed a land grant awarding territory to four companies. The following year, however, the legislature voided the law and declared all rights and claims under it to be invalid. In 1800, John Peck acquired land that was part of the original legislative grant. He then sold the land to Robert Fletcher three years later, claiming that past sales of the land had been legitimate. Fletcher argued that since the original sale of the land had been declared invalid, Peck had no legal right to sell the land and thus committed a breach of contract. Question Presented Could the contract between Fletcher and Peck be invalidated by an act of the Georgia legislature? Conclusion In a unanimous opinion, the Court held that since the estate had been legally "passed into the hands of a purchaser for a valuable consideration," the Georgia legislature could not take away the land or invalidate the contract. Noting that the Constitution did not permit bills of attainder or ex post facto laws, the Court held that laws annulling contracts or grants made by previous legislative acts were constitutionally impermissible.

*McCulloch v. Maryland (1819)

Facts of the Case In 1816, Congress chartered The Second Bank of the United States. In 1818, the state of Maryland passed legislation to impose taxes on the bank. James W. McCulloch, the cashier of the Baltimore branch of the bank, refused to pay the tax. Question Presented The case presented two questions: Did Congress have the authority to establish the bank? Did the Maryland law unconstitutionally interfere with congressional powers? Conclusion In a unanimous decision, the Court held that Congress had the power to incorporate the bank and that Maryland could not tax instruments of the national government employed in the execution of constitutional powers. Writing for the Court, Chief Justice Marshall noted that Congress possessed unenumerated powers not explicitly outlined in the Constitution. Marshall also held that while the states retained the power of taxation, "the constitution and the laws made in pursuance thereof are supreme. . .they control the constitution and laws of the respective states, and cannot be controlled by them."

*McCulloch v. Maryland (1819)

Facts of the Case In 1816, Congress chartered The Second Bank of the United States. In 1818, the state of Maryland passed legislation to impose taxes on the bank. James W. McCulloch, the cashier of the Baltimore branch of the bank, refused to pay the tax. Question Presented The case presented two questions: Did Congress have the authority to establish the bank? Did the Maryland law unconstitutionally interfere with congressional powers? Conclusion In a unanimous decision, the Court held that Congress had the power to incorporate the bank and that Maryland could not tax instruments of the national government employed in the execution of constitutional powers. Writing for the Court, Chief Justice Marshall noted that Congress possessed unenumerated powers not explicitly outlined in the Constitution. Marshall also held that while the states retained the power of taxation, "the constitution and the laws made in pursuance thereof are supreme. . .they control the constitution and laws of the respective states, and cannot be controlled by them."

Federal Election Commission v. Beaumont (2003)

Facts of the Case In 1971 Congress passed the Federal Election Campaign Act, banning direct corporate donations to federal election campaigns. In 2000, Christine Beaumont and the North Carolina Right to Life (NCRL), an anti-abortion advocacy group, challenged the act, saying it violated their right to free speech. The group is an incorporated non-profit that lobbies and backs political candidates friendly to its cause, but under the act it cannot make political donations. The district court ruled in favor of NCRL. The 4th Circuit Court of Appeals affirmed. Question Presented Does the Federal Election Campaign Act's ban on corporate political donations violate the freedom of speech for incorporated, non-profit advocacy groups? Conclusion No. In a 7-2 opinion delivered by Justice David H. Souter, the Court held that applying the direct contribution prohibition to nonprofit advocacy corporations is consistent with the First Amendment. The Court reasoned that it could not hold for NCRL "without recasting our understanding of the risks of harm posed by corporate political contributions, of the expressive significance of contributions, and of the consequent deference owed to legislative judgments on what to do about them." Justice Anthony M. Kennedy filed an opinion concurring in the judgment. Justice Clarence Thomas, joined by Justice Antonin Scalia, dissented, arguing that section 441b should have been subject to strict scrutiny and, under this standard, it could not stand

Federal Election Commission v. Beaumont (2003)

Facts of the Case In 1971 Congress passed the Federal Election Campaign Act, banning direct corporate donations to federal election campaigns. In 2000, Christine Beaumont and the North Carolina Right to Life (NCRL), an anti-abortion advocacy group, challenged the act, saying it violated their right to free speech. The group is an incorporated non-profit that lobbies and backs political candidates friendly to its cause, but under the act it cannot make political donations. The district court ruled in favor of NCRL. The 4th Circuit Court of Appeals affirmed. Question Presented Does the Federal Election Campaign Act's ban on corporate political donations violate the freedom of speech for incorporated, non-profit advocacy groups? Conclusion No. In a 7-2 opinion delivered by Justice David H. Souter, the Court held that applying the direct contribution prohibition to nonprofit advocacy corporations is consistent with the First Amendment. The Court reasoned that it could not hold for NCRL "without recasting our understanding of the risks of harm posed by corporate political contributions, of the expressive significance of contributions, and of the consequent deference owed to legislative judgments on what to do about them." Justice Anthony M. Kennedy filed an opinion concurring in the judgment. Justice Clarence Thomas, joined by Justice Antonin Scalia, dissented, arguing that section 441b should have been subject to strict scrutiny and, under this standard, it could not stand.

Texas v. Johnson (1989)

Facts of the Case In 1984, in front of the Dallas City Hall, Gregory Lee Johnson burned an American flag as a means of protest against Reagan administration policies. Johnson was tried and convicted under a Texas law outlawing flag desecration. He was sentenced to one year in jail and assessed a $2,000 fine. After the Texas Court of Criminal Appeals reversed the conviction, the case went to the Supreme Court. Question Presented Is the desecration of an American flag, by burning or otherwise, a form of speech that is protected under the First Amendment? Conclusion In a 5-to-4 decision, the Court held that Johnson's burning of a flag was protected expression under the First Amendment. The Court found that Johnson's actions fell into the category of expressive conduct and had a distinctively political nature. The fact that an audience takes offense to certain ideas or expression, the Court found, does not justify prohibitions of speech. The Court also held that state officials did not have the authority to designate symbols to be used to communicate only limited sets of messages, noting that "[i]f there is a bedrock principle underlying the First Amendment, it is that the Government may not prohibit the expression of an idea simply because society finds the idea itself offensive or disagreeable."

Gratz v. Bollinger (2003)

Facts of the Case In 1995, Jennifer Gratz applied to the University of Michigan's College of Literature, Science and the Arts with an adjusted GPA of 3.8 and ACT score of 25. In 1997, Patrick Hamacher applied to the University with an adjusted GPA of 3.0, and an ACT score of 28. Both were denied admission and attended other schools. The University admits that it uses race as a factor in making admissions decisions because it serves a "compelling interest in achieving diversity among its student body." In addition, the University has a policy to admit virtually all qualified applicants who are members of one of three select racial minority groups - African Americans, Hispanics, and Native Americans - that are considered to be "underrepresented" on the campus. Concluding that diversity was a compelling interest, the District Court held that the admissions policies for years 1995-1998 were not narrowly tailored, but that the policies in effect in 1999 and 2000 were narrowly tailored. After the decision in Grutter, Gratz and Hamacher petitioned the U.S. Supreme Court pursuant to Rule 11 for a writ of certiorari before judgment, which was granted. Question Presented Does the University of Michigan's use of racial preferences in undergraduate admissions violate the Equal Protection Clause of the Fourteenth Amendment or Title VI of the Civil Rights Act of 1964? Conclusion Yes. In a 6-3 opinion delivered by Chief Justice William H. Rehnquist, the Court held that the University of Michigan's use of racial preferences in undergraduate admissions violates both the Equal Protection Clause and Title VI. While rejecting the argument that diversity cannot constitute a compelling state interest, the Court reasoned that the automatic distribution of 20 points, or one-fifth of the points needed to guarantee admission, to every single "underrepresented minority" applicant solely because of race was not narrowly tailored and did not provide the individualized consideration Justice Powell contemplated in Regents of the University of California v. Baake, 438 U.S. 265 (1978). Chief Justice Rehnquist wrote, "because the University's use of race in its current freshman admissions policy is not narrowly tailored to achieve respondents' asserted compelling interest in diversity, the admissions policy violates the Equal Protection Clause."

Grutter v. Bollinger (2003)

Facts of the Case In 1997, Barbara Grutter, a white resident of Michigan, applied for admission to the University of Michigan Law School. Grutter applied with a 3.8 undergraduate GPA and an LSAT score of 161. She was denied admission. The Law School admits that it uses race as a factor in making admissions decisions because it serves a "compelling interest in achieving diversity among its student body." The District Court concluded that the Law School's stated interest in achieving diversity in the student body was not a compelling one and enjoined its use of race in the admissions process. In reversing, the Court of Appeals held that Justice Powell's opinion in Regents of the University of California v. Bakke, 438 U.S. 265 (1978), constituted a binding precedent establishing diversity as a compelling governmental interest sufficient under strict scrutiny review to justify the use of racial preferences in admissions. The appellate court also rejected the district court's finding that the Law School's "critical mass" was the functional equivalent of a quota. Question Presented Does the University of Michigan Law School's use of racial preferences in student admissions violate the Equal Protection Clause of the Fourteenth Amendment or Title VI of the Civil Rights Act of 1964? Conclusion No. In a 5-4 opinion delivered by Justice Sandra Day O'Connor, the Court held that the Equal Protection Clause does not prohibit the Law School's narrowly tailored use of race in admissions decisions to further a compelling interest in obtaining the educational benefits that flow from a diverse student body. The Court reasoned that, because the Law School conducts highly individualized review of each applicant, no acceptance or rejection is based automatically on a variable such as race and that this process ensures that all factors that may contribute to diversity are meaningfully considered alongside race. Justice O'Connor wrote, "in the context of its individualized inquiry into the possible diversity contributions of all applicants, the Law School's race-conscious admissions program does not unduly harm nonminority applicants."

*Near v. Minnesota ex rel. Olson (1931) (repeat from Incorporation)

Facts of the Case Jay Near published a scandal sheet in Minneapolis, in which he attacked local officials, charging that they were implicated with gangsters. Minnesota officials obtained an injunction to prevent Near from publishing his newspaper under a state law that allowed such action against periodicals. The law provided that any person "engaged in the business" of regularly publishing or circulating an "obscene, lewd, and lascivious" or a "malicious, scandalous and defamatory" newspaper or periodical was guilty of a nuisance, and could be enjoined (stopped) from further committing or maintaining the nuisance. Question Presented Does the Minnesota "gag law" violate the free press provision of the First Amendment? Conclusion The Supreme Court held that the statute authorizing the injunction was unconstitutional as applied. History had shown that the protection against previous restraints was at the heart of the First Amendment. The Court held that the statutory scheme constituted a prior restraint and hence was invalid under the First Amendment. Thus the Court established as a constitutional principle the doctrine that, with some narrow exceptions, the government could not censor or otherwise prohibit a publication in advance, even though the communication might be punishable after publication in a criminal or other proceeding.

Near v. Minnesota ex rel. Olson (1931)

Facts of the Case Jay Near published a scandal sheet in Minneapolis, in which he attacked local officials, charging that they were implicated with gangsters. Minnesota officials obtained an injunction to prevent Near from publishing his newspaper under a state law that allowed such action against periodicals. The law provided that any person "engaged in the business" of regularly publishing or circulating an "obscene, lewd, and lascivious" or a "malicious, scandalous and defamatory" newspaper or periodical was guilty of a nuisance, and could be enjoined (stopped) from further committing or maintaining the nuisance. Question Presented Does the Minnesota "gag law" violate the free press provision of the First Amendment? Conclusion The Supreme Court held that the statute authorizing the injunction was unconstitutional as applied. History had shown that the protection against previous restraints was at the heart of the First Amendment. The Court held that the statutory scheme constituted a prior restraint and hence was invalid under the First Amendment. Thus the Court established as a constitutional principle the doctrine that, with some narrow exceptions, the government could not censor or otherwise prohibit a publication in advance, even though the communication might be punishable after publication in a criminal or other proceeding.

Barron v. Mayor and City Council of Baltimore (1833)

Facts of the Case John Barron was co-owner of a profitable wharf in the harbor of Baltimore. As the city developed and expanded, large amounts of sand accumulated in the harbor, depriving Barron of the deep waters which had been the key to his successful business. He sued the city to recover a portion of his financial losses. Question Presented Does the Fifth Amendment deny the states as well as the national government the right to take private property for public use without justly compensating the property's owner? Conclusion No. The Court announced its decision in this case without even hearing the arguments of the City of Baltimore. Writing for the unanimous Court, Chief Justice Marshall found that the limitations on government articulated in the Fifth Amendment were specifically intended to limit the powers of the national government. Citing the intent of the framers and the development of the Bill of Rights as an exclusive check on the government in Washington D.C., Marshall argued that the Supreme Court had no jurisdiction in this case since the Fifth Amendment was not applicable to the states.

Lawrence and Garner v. Texas (2003)

Facts of the Case Responding to a reported weapons disturbance in a private residence, Houston police entered John Lawrence's apartment and saw him and another adult man, Tyron Garner, engaging in a private, consensual sexual act. Lawrence and Garner were arrested and convicted of deviate sexual intercourse in violation of a Texas statute forbidding two persons of the same sex to engage in certain intimate sexual conduct. In affirming, the State Court of Appeals held that the statute was not unconstitutional under the Due Process Clause of the Fourteenth Amendment, with Bowers v. Hardwick, 478 U.S. 186 (1986), controlling. Question Presented Do the criminal convictions of John Lawrence and Tyron Garner under the Texas "Homosexual Conduct" law, which criminalizes sexual intimacy by same-sex couples, but not identical behavior by different-sex couples, violate the Fourteenth Amendment guarantee of equal protection of laws? Do their criminal convictions for adult consensual sexual intimacy in the home violate their vital interests in liberty and privacy protected by the Due Process Clause of the Fourteenth Amendment? Should Bowers v. Hardwick, 478 U.S. 186 (1986), be overruled? Conclusion No, yes, and yes. In a 6-3 opinion delivered by Justice Anthony M. Kennedy, the Court held that the Texas statute making it a crime for two persons of the same sex to engage in certain intimate sexual conduct violates the Due Process Clause. After explaining what it deemed the doubtful and overstated premises of Bowers, the Court reasoned that the case turned on whether Lawrence and Garner were free as adults to engage in the private conduct in the exercise of their liberty under the Due Process Clause. "Their right to liberty under the Due Process Clause gives them the full right to engage in their conduct without intervention of the government," wrote Justice Kennedy. "The Texas statute furthers no legitimate state interest which can justify its intrusion into the personal and private life of the individual," continued Justice Kennedy. Accordingly, the Court overruled Bowers. Justice Sandra Day O'Connor filed an opinion concurring in the judgment. Justices Clarence Thomas and Antonin Scalia, with whom Chief Justice William H. Rehnquist and Justices Thomas joined, filed dissents.

Reno v. ACLU (1997)

Facts of the Case Several litigants challenged the constitutionality of two provisions in the 1996 Communications Decency Act. Intended to protect minors from unsuitable internet material, the Act criminalized the intentional transmission of "obscene or indecent" messages as well as the transmission of information which depicts or describes "sexual or excretory activities or organs" in a manner deemed "offensive" by community standards. After being enjoined by a District Court from enforcing the above provisions, except for the one concerning obscenity and its inherent protection against child pornography, Attorney General Janet Reno appealed directly to the Supreme Court as provided for by the Act's special review provisions. Question Presented Did certain provisions of the 1996 Communications Decency Act violate the First and Fifth Amendments by being overly broad and vague in their definitions of the types of internet communications which they criminalized? Conclusion Yes. The Court held that the Act violated the First Amendment because its regulations amounted to a content-based blanket restriction of free speech. The Act failed to clearly define "indecent" communications, limit its restrictions to particular times or individuals (by showing that it would not impact on adults), provide supportive statements from an authority on the unique nature of internet communications, or conclusively demonstrate that the transmission of "offensive" material is devoid of any social value. The Court added that since the First Amendment distinguishes between "indecent" and "obscene" sexual expressions, protecting only the former, the Act could be saved from facial overbreadth challenges if it dropped the words "or indecent" from its text. The Court refused to address any Fifth Amendment issues

Boy Scouts of America v. Dale (2000)

Facts of the Case The Boy Scouts of America revoked former Eagle Scout and assistant scoutmaster James Dale's adult membership when the organization discovered that Dale was a homosexual and a gay rights activist. In 1992, Dale filed suit against the Boy Scouts, alleging that the Boy Scouts had violated the New Jersey statute prohibiting discrimination on the basis of sexual orientation in places of public accommodation. The Boy Scouts, a private, not-for-profit organization, asserted that homosexual conduct was inconsistent with the values it was attempting to instill in young people. The New Jersey Superior Court held that New Jersey's public accommodations law was inapplicable because the Boy Scouts was not a place of public accommodation. The court also concluded that the Boy Scouts' First Amendment freedom of expressive association prevented the government from forcing the Boy Scouts to accept Dale as an adult leader. The court's Appellate Division held that New Jersey's public accommodations law applied to the Boy Scouts because of its broad-based membership solicitation and its connections with various public entities, and that the Boy Scouts violated it by revoking Dale's membership based on his homosexuality. The court rejected the Boy Scouts' federal constitutional claims. The New Jersey Supreme Court affirmed. The court held that application of New Jersey's public accommodations law did not violate the Boy Scouts' First Amendment right of expressive association because Dale's inclusion would not significantly affect members' abilities to carry out their purpose. Furthermore, the court concluded that reinstating Dale did not compel the Boy Scouts to express any message. Question Presented Does the application of New Jersey's public accommodations law violate the Boy Scouts' First Amendment right of expressive association to bar homosexuals from serving as troop leaders? Conclusion Yes. In a 5-4 opinion delivered by Chief Justice William H. Rehnquist, the Court held that "applying New Jersey's public accommodations law to require the Boy Scouts to admit Dale violates the Boy Scouts' First Amendment right of expressive association." In effect, the ruling gives the Boy Scouts of America a constitutional right to bar homosexuals from serving as troop leaders. Chief Justice Rehnquist wrote for the Court that, "[t]he Boy Scouts asserts that homosexual conduct is inconsistent with the values it seeks to instill," and that a gay troop leader's presence "would, at the very least, force the organization to send a message, both to the young members and the world, that the Boy Scouts accepts homosexual conduct as a legitimate form of behavior."

*The Civil Rights Cases (1883)

Facts of the Case The Civil Rights Act of 1875 affirmed the equality of all persons in the enjoyment of transportation facilities, in hotels and inns, and in theaters and places of public amusement. Though privately owned, these businesses were like public utilities, exercising public functions for the benefit of the public and, thus, subject to public regulation. In five separate cases, a black person was denied the same accommodations as a white person in violation of the 1875 Act. Question Presented Does the Civil Rights Act of 1875 violate the 10th Amendment of the Constitution which reserves all powers not granted to the national government to the states or to the people? Conclusion The Fourteenth Amendment restrains only state action. And the fifth section of the Amendment empowers Congress only to enforce the prohibition on state action. The amendment did not authorize national legislation on subjects which are within the domain of the state. Private acts of racial discrimination were simply private wrongs that the national government was powerless to correct.

Hazelwood School District v. Kuhlmeier (1988)

Facts of the Case The Spectrum, the school-sponsored newspaper of Hazelwood East High School, was written and edited by students. In May 1983, Robert E. Reynolds, the school principal, received the pages proofs for the May 13 issue. Reynolds found two of the articles in the issue to be inappropriate, and ordered that the pages on which the articles appeared be withheld from publication. Cathy Kuhlmeier and two other former Hazelwood East students brought the case to court. Question Presented Did the principal's deletion of the articles violate the students' rights under the First Amendment? Conclusion No. In a 5-to-3 decision, the Court held that the First Amendment did not require schools to affirmatively promote particular types of student speech. The Court held that schools must be able to set high standards for student speech disseminated under their auspices, and that schools retained the right to refuse to sponsor speech that was "inconsistent with 'the shared values of a civilized social order.'" Educators did not offend the First Amendment by exercising editorial control over the content of student speech so long as their actions were "reasonably related to legitimate pedagogical concerns." The actions of principal Reynolds, the Court held, met this test.

Locke v. Davey (2004)

Facts of the Case The Washington State Promise Scholarship, created by the state legislature in 1999, gives college scholarship money to talented students. However, this money cannot be used to obtain a degree in theology if the program is taught to cause belief. Washington's constitution prohibits funding religious instruction. The 1969 state code applied this ban to college financial aid. Joshua Davey forfeited his Promise Scholarship money in order to major in pastoral ministries at a private Christian college. Davey filed suit in U.S. district court, claiming the state constitution's ban on funding religious instruction violated his First Amendment right to free exercise of religion (in the U.S. Constitution). The district court rejected Davey's claim. The Ninth Circuit Court of Appeals reversed, concluding Davey's free exercise rights were violated. Question Presented If a state provides college scholarships for secular instruction, does the First Amendment's free exercise clause require a state to fund religious instruction? Conclusion (from another site) On February 25, 2004, the United States Supreme Court announced its ruling in Locke v. Davey, holding that Washington State is allowed to deny scholarship funds to students studying devotional theology. Justice Rehnquist wrote for the majority: "The State of Washington established the Promise Scholarship Program to assist academically gifted students with postsecondary education expenses. In accordance with the State Constitution, students may not use the scholarship at an institution where they are pursuing a degree in devotional theology. We hold that such an exclusion from an otherwise inclusive aid program does not violate the Free Exercise Clause of the First Amendment."

*Marbury v. Madison (1803)

Facts of the Case The case began on March 2, 1801, when an obscure Federalist, William Marbury, was designated as a justice of the peace in the District of Columbia. Marbury and several others were appointed to government posts created by Congress in the last days of John Adams's presidency, but these last-minute appointments were never fully finalized. The disgruntled appointees invoked an act of Congress and sued for their jobs in the Supreme Court. Question Presented Is Marbury entitled to his appointment? Is his lawsuit the correct way to get it? And, is the Supreme Court the place for Marbury to get the relief he requests? Conclusion Yes; yes; and it depends. The justices held, through Marshall's forceful argument, that on the last issue the Constitution was "the fundamental and paramount law of the nation" and that "an act of the legislature repugnant to the constitution is void." In other words, when the Constitution--the nation's highest law--conflicts with an act of the legislature, that act is invalid. This case establishes the Supreme Court's power of judicial review. The decision in Marbury v. Madison is a landmark case because it created the concept of "judicial review," which allows the Supreme Court to declare the actions of Congress unconstitutional. The case arose from the defeat of President Adams in the 1800 election. Adams appointed many Federalists, members of his party, to federal court positions the night before he was to give up his office; however, the appointments were never delivered when President Jefferson's administration took control. Chief Justice Marshall took this oppotunity to more exactly define the Supreme Court's powers. He explained that Congress could not expand or contract the Supreme Court's original jurisidiction as stated in the Constitution, as Congress attempted to in the Judiciary Act of 1789, where it authorized the Court to issue writs of mandamus ordering federal officers to perform certain tasks. Thus the Supreme Court was not allowed to order the president to deliver the appointments. This decision did, however, set the precedent that the Supreme Court could declare Acts of Congress unconstitutional.

*Plessy v. Ferguson (1896)

Facts of the Case The state of Louisiana enacted a law that required separate railway cars for blacks and whites. In 1892, Homer Adolph Plessy--who was seven-eighths Caucasian--took a seat in a "whites only" car of a Louisiana train. He refused to move to the car reserved for blacks and was arrested. Question Presented Is Louisiana's law mandating racial segregation on its trains an unconstitutional infringement on both the privileges and immunities and the equal protection clauses of the Fourteenth Amendment? Conclusion No, the state law is within constitutional boundaries. The majority upheld state-imposed racial segregation. The justices based their decision on the separate-but-equal doctrine, that separate facilities for blacks and whites satisfied the Fourteenth Amendment so long as they were equal. (The phrase, "separate but equal" was not part of the opinion.) Brown conceded that the 14th amendment intended to establish absolute equality for the races before the law. But Brown noted that "in the nature of things it could not have been intended to abolish distinctions based upon color, or to enforce social, as distinguished from political equality, or a commingling of the two races unsatisfactory to either." In short, segregation does not in itself constitute unlawful discrimination.

* NLRB ( National Labor Relations Board) v. Jones & Laughlin Steel Corp. (1937)

Facts of the Case With the National Labor Relations Act of 1935, Congress determined that labor-management disputes were directly related to the flow of interstate commerce and, thus, could be regulated by the national government. In this case, the National Labor Relations Board charged the Jones & Laughlin Steel Co. with discriminating against employees who were union members. Question Presented Was the Act consistent with the Commerce Clause? Conclusion Yes. The Court held that the Act was narrowly constructed so as to regulate industrial activities which had the potential to restrict interstate commerce. The justices abandoned their claim that labor relations had only an indirect effect on commerce. Since the ability of employees to engage in collective bargaining (one activity protected by the Act) is "an essential condition of industrial peace," the national government was justified in penalizing corporations engaging in interstate commerce which "refuse to confer and negotiate" with their workers. *

Fay v. Noia 1963

Fay v. Noia illustrates the ambiguity of the federal and state appellate processes. Noia had been convicted of a felony murder in the state of New York. A questionable confession had been offered into evidence by the state which Noia wanted to appeal. The time having elapsed for him to appeal to the state, he filed a writ of habeas corpus in the Supreme Court. The law, however, stated that a person must exhaust all state courts before appealing to a federal court. Because of this, the state of New York admitted the confession was coerced, but argued that the appeal for it was inadmissible. The Supreme Court found in favor of Noia; Justice William Brennan asserted that the government had few duties to its citizens as important as the maintenence of their right to seek habeas corpus, which is based on the firm "principle... that in a civilized society, government must always be accountable to the judiciary for a man's imprisonment."

Gregg v. Georgia (1976)

In this case the court upheld the constitutionality of the death penalty, defending statutes that guide judges and juries in the decision to issue the death sentence. The Court did, however, state that the mandatory use of the death penalty would be prohibited under the Eighth Amendment as cruel and unusual punishment. The defendent in this case, Gregg, had been convicted on two counts of armed robbery and two counts of murder. The jury was instructed by the trial judge, who was following Georgia state law, to return with either a decision of life imprisonment or the death penalty. Justice Byron stated in his opinion that Gregg had failed in his burden of showing that the Georgia Supreme Court had not done all it could to prevent discriminatory practices in the forming of his sentence. This decision became the first time the Court stated that "punishment of death does not invariably violate the Constitution."

Feiner v. New York (1951)

Irving Feiner was arrested on the evening of March 8th, 1949, for disorderly conduct. Feiner had been speaking out against President Truman, the American Legion, and local officials atop a wooden box on a street corner. A crowd of between seventy and eighty people gathered to watch as he urged blacks to violently oppose the injustices of society. The crowd was of a mixed opinion and became more and more unruly, until the police arrested Feiner and broke up the gathering. The police, prior to the arrest, had asked him three times to get off the podium, and he had refused each time, claiming his First Amendment right to free speech. Feiner was convicted and appealed the decision to the Supreme Court. The Court ruled that the arrest did not hamper Feiner's right to free speech, stating "it is one thing to say that the police cannot be used as an instrument of suppression of unpopular views, and another to say that...they are powerless to prevent a breach of the peace." The Court had limited free speech to all but those instances when there was clear and present danger.

Katz v. United States (7-1 vote, December 18, 1967)

Katz was arrested for illegal gambling after using a public phone to transmit "gambling information." The FBI had attached an electronic listening/recording device onto the outside of the public phone booth that Katz habitually used. They argued that this constituted a legal action since they never actually entered the phone booth. The Court, however, ruled in favor of Katz, stating the Fourth Amendment allowed for the protection of a person and not just a person's property against illegal searches. Whatever a citizen "seeks to preserve as private, even in an area accesible to the public, may be constitutionally protected."

Texas v. Johnson (1989)

Outside the Republican National Convention in Dallas, a protest of Ronald Reagan's policies had been organized, during which a United States flag was burned. Johnson, the man responsible for the flag burning, was arrested under Texas law, which made the desecration of the United States or Texas flags crimes. Johnson was convicted and sentenced to one year in jail and a two thousand dollar fine. Texas reasoned that the police were preventing the breach of peace that would be erupt due to the flag burning, and preserving the integrity of the flag as a symbol of national unity. Johnson's conviction was overturned by the Supreme Court of Texas, which ruled that this mode of self-expression was protected under the First Amendment to the Constitution. The Supreme Court upheld this ruling, stating the flag burning was "expressive conduct" because it was an attempt to "convey a particularized message."

Miranda v. Arizona (1966)

Miranda is one of the best-known cases in the history of the Supreme Court. It represents the Court's determination to treat even the lowliest of criminals with the same dignity and respect as the wealthiest celebrity. Ernesto Miranda was arrested in Phoenix and taken directly to the police station. A victim of rape and kidnapping identified him as the perpetrator. The police then brought Miranda into the interrogation room, questioned him for two hours, and received a signed confession. The police had never advised Miranda of his right to an attorney or the fact that anything he said could be used against him in a court of law. Although Miranda's treatment was actually quite mild, compared to some of the other methods used at the time, the Supreme Court still found in favor of him, holding that "the defendant's confession was inadmissible because he was not in any way [informed] of his right to council nor was his privilege against self-incrimination effectively protected in any other manner."

Powell v. Alabama (1932)

Nine black youths were arrested and charged with the rape of a white woman in Scottsboro, Alabama. Powell v. Alabama was the first of the ensuing rape trials. Powell's argument addressed the lack of legal representation he was given. Upon the day of the trial, the attorney appointed to his case refused to represent him; when the judge ordered any attorney present in the courtroom to serve as the boys' legal council, they all refused. Once council willing to represent him was found, the attorney had no time to review the case, and only a half hour to meet with his client before the hearing. As a result of this inadequate defense, Powell was sentenced to die along with his eight friends. The Supreme Court reversed the verdict because of its belief that poor people facing the death penalty must be provided with council, stating "...there is a certain principals of Justice which adhere to the very idea of free government, which no [State] may disregard." This decision was extended to include all people facing criminal proceedings in Gideon v. Wainwright (1963).

Olmstead v. United States (5-4 vote, decided June 4, 1928)

Olmstead was convicted for the illegal sale of alcohol under the National Prohibition Act. Much of the evidence used against him was obtained through the use of an illegal wiretap placed on his phone. Olmstead argued to the Supreme Court that his Fourth Amendment rights allowed him a "reasonable expectation of privacy." The Court disagreed, stating that Olmstead intended "to project his voice to those quite outside...and that...nothing tangible was taken." This verdict was later overturned when, in 1934, the United States Congress enacted the Federal Communications Act. The Act prohibited the interception of any communications and the divulgence of the contents of intercepted communications. The Court then overturned the Olmstead decision, extending the exclusionary rule to include wiretapping in federal prosecutions

Mapp v. Ohio 5-3 vote, (June 19, 1961)

One night, seven police officers broke into and searched Dolly Mapp's home in Cleveleand, Ohio. The search was prompted by an informant telling them that a suspect in a recent bombing was hiding out there. Mapp's house was searched, and no sign of the supsect was found; however, police did find some literature deemed obscene (the possession of which was a crime). Although the police claimed to have a search warrant, but none was produced. In court, their search was upheld and Dolly Mapp was convicted of the possession of obscene material. The Supreme Court overturned this conviction on the grounds that the search was illegal. The Mapp case incorporated the 4th amendment into the Due Process clause of the 14th amendment and created the "exclusionary rule," which prevents the use of evidence gained by these so-called illegal searches. Opponents of the exclusionary rule argue "the criminal is to go free because the constable has blundered," to which Justice Clark answered "The criminal goes free if he must, but it is the law that sets him free."

Baker v. Carr (1962)

Population was not always the basis for the apportionment of seats to the state legislatures. Rather, the states were divided into districts based on land area. Originally, this gave a slight advantage to citizens of rural districts. With less people living in them, each person's vote had greater weight. With the growth of cities, this advantage had grown to a huge disproportion by the 1960s. Justice Tom C. Clark cited in his opinion that two-thirds of the members of the state Senate were elected by one-third of the population and two-thirds of the members of the House were elected by forty percent of the population. This was most noticable in the more urbanized areas. It was this inequality in representation that motivated the "city dwellers" to bring a suit to the federal courts against their states on the grounds that the unequal distribution of "voting power" infringed on their 14th Amendment right to equal protection under the law. The federal courts ruled that they had no jurisdiction over this legislative matter. The Supreme Court then overturned this decision and ruled only that this matter was "justicable" shattering the idea that there were political questions outside the courts jurisdiction

Nix v. Williams (1984)

Robert Williams was arrested for the murder of a 10-year-old girl. He was never advised of his rights or allowed to speak with council. The police asked him for the location of the victim's body in order to give it a proper Christian burial. Williams agreed and showed them where they could find it. Williams was convicted, and the body was offered as the main piece of evidence. William's attorney protested the use of the body stating that the police had acted in bad faith, forcing the whereabouts of the body from the defendant. The Court disagreed, seeing no bad faith on the part of the police. It allowed the body into evidence on the grounds that the police would have found it regardless of William's admission, due to the fact search parties were within two miles of it when the location was revealed. "Evidence otherwise excluded may be admissible when it would have been discovered anyway."

Adarand Constructors v. Pena (1995)

Subjects: Civil Rights: Affirmative Action Facts of the Case Adarand, a contractor specializing in highway guardrail work, submitted the lowest bid as a subcontractor for part of a project funded by the United States Department of Transportation. Under the terms of the federal contract, the prime contractor would receive additional compensation if it hired small businesses controlled by "socially and economically disadvantaged individuals." [The clause declared that "the contractor shall presume that socially and economically disadvantaged individuals include Black Americans, Hispanic Americans, Native Americans, Asian Pacific Americans, and other minorities...." Federal law requires such a subcontracting clause in most federal agency contracts]. Another subcontractor, Gonzales Construction Company, was awarded the work. It was certified as a minority business; Adarand was not. The prime contractor would have accepted Adarand's bid had it not been for the additional payment for hiring Gonzales. Question Presented Is the presumption of disadvantage based on race alone, and consequent allocation of favored treatment, a discriminatory practice that violates the Fifth Amendment's Equal Protection Clause? Conclusion Yes. Overruling Metro Broadcasting (497 US 547), the Court held that all racial classifications, whether imposed by federal, state, or local authorities, must pass strict scrutiny review. In other words, they "must serve a compelling government interest, and must be narrowly tailored to further that interest." The Court added that compensation programs which are truly based on disadvantage, rather than race, would be evaluated under lower equal protection standards. However, since race is not a sufficient condition for a presumption of disadvantage and the award of favored treatment, all race-based classifications must be judged under the strict scrutiny standard. Moreover, even proof of past injury does not in itself establish the suffering of present or future injury. The Court remanded for a determination of whether the Transportation Department's program satisfied strict scrutiny.

*Regents of the University of California v. Bakke (1978)

Subjects: Civil Rights: Affirmative Action Facts of the Case Allan Bakke, a thirty-five-year-old white man, had twice applied for admission to the University of California Medical School at Davis. He was rejected both times. The school reserved sixteen places in each entering class of one hundred for "qualified" minorities, as part of the university's affirmative action program, in an effort to redress longstanding, unfair minority exclusions from the medical profession. Bakke's qualifications (college GPA and test scores) exceeded those of any of the minority students admitted in the two years Bakke's applications were rejected. Bakke contended, first in the California courts, then in the Supreme Court, that he was excluded from admission solely on the basis of race. Question Presented Did the University of California violate the Fourteenth Amendment's equal protection clause, and the Civil Rights Act of 1964, by practicing an affirmative action policy that resulted in the repeated rejection of Bakke's application for admission to its medical school? Conclusion No and yes. There was no single majority opinion. Four of the justices contended that any racial quota system supported by government violated the Civil Rights Act of 1964. Justice Lewis F. Powell, Jr., agreed, casting the deciding vote ordering the medical school to admit Bakke. However, in his opinion, Powell argued that the rigid use of racial quotas as employed at the school violated the equal protection clause of the Fourteenth Amendment. The remaining four justices held that the use of race as a criterion in admissions decisions in higher education was constitutionally permissible. Powell joined that opinion as well, contending that the use of race was permissible as one of several admission criteria. So, the Court managed to minimize white opposition to the goal of equality (by finding for Bakke) while extending gains for racial minorities through affirmative action.

Katzenbach v. McClung (1964)

Subjects: Civil Rights: Desegregation Facts of the Case The owner of Ollie's Barbecue, in Birmingham Alabama, refused to serve blacks in apparent violation of the Civil Rights Act of 1964. Part of the Act prevented restaurants serving interstate travelers, or receiving a substantial amount of their food from interstate commerce, from discriminating on the basis of race. Question Presented Does a restaurant's refusal to serve blacks burden interstate commerce to an extent that Congress can legitimately prohibit such discrimination? Conclusion The Court found that discrimination in restaurants posed significant burdens on "the interstate flow of food and upon the movement on products generally." Furthermore, argued Justice Clark, discrimination also posed restrictions on blacks who traveled from state to state. Congress's solution to this problem was appropriate and within its bounds to regulate interstate commerce.

*Heart of Atlanta Motel v. U.S. (1964)

Subjects: Civil Rights: Desegregation Facts of the Case Title II of the Civil Rights Act of 1964 forbade racial discrimination by places of public accommodation if their operations affected commerce. The Heart of Atlanta Motel in Atlanta, Georgia, refused to accept Black Americans and was charged with violating Title II. Question Presented Did Congress, in passing Title II of the 1964 Civil Rights Act, exceed its Commerce Clause powers by depriving motels, such as the Heart of Atlanta, of the right to choose their own customers? Conclusion The Court held that the Commerce Clause allowed Congress to regulate local incidents of commerce, and that the Civil Right Act of 1964 passed constitutional muster. The Court noted that the applicability of Title II was "carefully limited to enterprises having a direct and substantial relation to the interstate flow of goods and people. . ." The Court thus concluded that places of public accommodation had no "right" to select guests as they saw fit, free from governmental regulation.

Swann v. Charlotte-Mecklenburg Bd. of Ed. (1971)

Subjects: Civil Rights: Desegregation, Schools Facts of the Case After the Supreme Court's decision in 1954 in Brown v. Board of Education, little progress had been made in desegregating public schools. One example was the Charlotte-Mecklenburg, North Carolina, system in which approximately 14,000 black students attended schools that were either totally black or more than 99 percent black. Lower courts had experimented with a number of possible solutions when the case reached the Supreme Court. Question Presented Were federal courts constitutionally authorized to oversee and produce remedies for state-imposed segregation? Conclusion In a unanimous decision, the Court held that once violations of previous mandates directed at desegregating schools had occurred, the scope of district courts' equitable powers to remedy past wrongs were broad and flexible. The Court ruled that 1) remedial plans were to be judged by their effectiveness, and the use of mathematical ratios or quotas were legitimate "starting points" for solutions; 2) predominantly or exclusively black schools required close scrutiny by courts; 3) non-contiguous attendance zones, as interim corrective measures, were within the courts' remedial powers; and 4) no rigid guidelines could be established concerning busing of students to particular schools.

*Brown v. Board of Education of Topeka (1954)

Subjects: Civil Rights: Desegregation, Schools Facts of the Case Black children were denied admission to public schools attended by white children under laws requiring or permitting segregation according to the races. The white and black schools approached equality in terms of buildings, curricula, qualifications, and teacher salaries. This case was decided together with Briggs v. Elliott and Davis v. County School Board of Prince Edward County. Question Presented Does the segregation of children in public schools solely on the basis of race deprive the minority children of the equal protection of the laws guaranteed by the 14th Amendment? Conclusion Yes. Despite the equalization of the schools by "objective" factors, intangible issues foster and maintain inequality. Racial segregation in public education has a detrimental effect on minority children because it is interpreted as a sign of inferiority. The long-held doctrine that separate facilities were permissible provided they were equal was rejected. Separate but equal is inherently unequal in the context of public education. The unanimous opinion sounded the death-knell for all forms of state-maintained racial separation.

U.S. Term Limits v. Thornton (1995)

Subjects: Federalism: Miscellaneous Federalism Facts of the Case On November 3, 1992, Arkansas voters adopted Amendment 73 to their State Constitution. The "Term Limitation Amendment," in addition to limiting terms of elected officials within the Arkansas state government, also provided that any person who served three or more terms as a member of the United States House of Representatives from Arkansas would be ineligible for re-election as a US Representative from Arkansas. Similarly, the Amendment provided that any person who served two or more terms as a member of the United States Senate from Arkansas would be ineligible for re-election as a US Senator from Arkansas. Question Presented Can states alter those qualifications for the U.S. Congress that are specifically enumerated in the Constitution? Conclusion No. The Constitution prohibits States from adopting Congressional qualifications in addition to those enumerated in the Constitution. A state congressional term limits amendment is unconstitutional if it has the likely effect of handicapping a class of candidates and "has the sole purpose of creating additional qualifications indirectly." Furthermore, "...allowing individual States to craft their own congressional qualifications would erode the structure designed by the Framers to form a 'more perfect Union.'"

* Shaw v. Reno (1993)

Subjects: Civil Rights: Reapportionment Facts of the Case The U.S. Attorney General rejected a North Carolina congressional reapportionment plan because the plan created only one black-majority district. North Carolina submitted a second plan creating two black-majority districts. One of these districts was, in parts, no wider than the interstate road along which it stretched. Five North Carolina residents challenged the constitutionality of this unusually shaped district, alleging that its only purpose was to secure the election of additional black representatives. After a three-judge District Court ruled that they failed to state a constitutional claim, the residents appealed and the Supreme Court granted certiorari. Question Presented Did the North Carolina residents' claim, that the State created a racially gerrymandered district, raise a valid constitutional issue under the Fourteenth Amendment's Equal Protection Clause? Conclusion Yes. The Court held that although North Carolina's reapportionment plan was racially neutral on its face, the resulting district shape was bizarre enough to suggest that it constituted an effort to separate voters into different districts based on race. The unusual district, while perhaps created by noble intentions, seemed to exceed what was reasonably necessary to avoid racial imbalances. After concluding that the residents' claim did give rise to an equal protection challenge, the Court remanded - adding that in the absence of contradictory evidence, the District Court would have to decide whether or not some compelling governmental interest justified North Carolina's plan.

Reed v. Reed (1971)

Subjects: Civil Rights: Sex Discrimination Facts of the Case The Idaho Probate Code specified that "males must be preferred to females" in appointing administrators of estates. After the death of their adopted son, both Sally and Cecil Reed sought to be named the administrator of their son's estate (the Reeds were separated). According to the Probate Code, Cecil was appointed administrator and Sally challenged the law in court. Question Presented Did the Idaho Probate Code violate the Equal Protection Clause of the Fourteenth Amendment? Conclusion In a unanimous decision, the Court held that the law's dissimilar treatment of men and women was unconstitutional. The Court argued that "[t]o give a mandatory preference to members of either sex over members of the other, merely to accomplish the elimination of hearings on the merits, is to make the very kind of arbitrary legislative choice forbidden by the Equal Protection Clause of the Fourteenth Amendment. . .[T]he choice in this context may not lawfully be mandated solely on the basis of sex."

*United States v. Nixon (1974)

Subjects: Criminal Procedure: Discovery and Inspection Facts of the Case A grand jury returned indictments against seven of President Richard Nixon's closest aides in the Watergate affair. The special prosecutor appointed by Nixon and the defendants sought audio tapes of conversations recorded by Nixon in the Oval Office. Nixon asserted that he was immune from the subpoena claiming "executive privilege," which is the right to withhold information from other government branches to preserve confidential communications within the executive branch or to secure the national interest. Decided together with Nixon v. United States. Question Presented Is the President's right to safeguard certain information, using his "executive privilege" confidentiality power, entirely immune from judicial review? Conclusion No. The Court held that neither the doctrine of separation of powers, nor the generalized need for confidentiality of high-level communications, without more, can sustain an absolute, unqualified, presidential privilege. The Court granted that there was a limited executive privilege in areas of military or diplomatic affairs, but gave preference to "the fundamental demands of due process of law in the fair administration of justice." Therefore, the president must obey the subpoena and produce the tapes and documents. Nixon resigned shortly after the release of the tapes.

* United States v. Lopez (1995)

Subjects: Economic Activity: Misc. Economic Regulation Facts of the Case Alfonzo Lopez, a 12th grade high school student, carried a concealed weapon into his San Antonio, Texas high school. He was charged under Texas law with firearm possession on school premises. The next day, the state charges were dismissed after federal agents charged Lopez with violating a federal criminal statute, the Gun-Free School Zones Act of 1990. The act forbids "any individual knowingly to possess a firearm at a place that [he] knows...is a school zone." Lopez was found guilty following a bench trial and sentenced to six months' imprisonment and two years' supervised release. Question Presented Is the 1990 Gun-Free School Zones Act, forbidding individuals from knowingly carrying a gun in a school zone, unconstitutional because it exceeds the power of Congress to legislate under the Commerce Clause? Conclusion Yes. The possession of a gun in a local school zone is not an economic activity that might, through repetition elsewhere, have a substantial effect on interstate commerce. The law is a criminal statute that has nothing to do with "commerce" or any sort of economic activity.

*Printz v. United States (1997)

Subjects: Federalism: Miscellaneous Facts of the Case The Brady Handgun Violence Prevention Act (Brady Bill) required "local chief law enforcement officers" (CLEOs) to perform background-checks on prospective handgun purchasers, until such time as the Attorney General establishes a federal system for this purpose. County sheriffs Jay Printz and Richard Mack, separately challenged the constitutionality of this interim provision of the Brady Bill on behalf of CLEOs in Montana and Arizona respectively. In both cases District Courts found the background-checks unconstitutional, but ruled that since this requirement was severable from the rest of the Brady Bill a voluntary background-check system could remain. On appeal from the Ninth Circuit's ruling that the interim background-check provisions were constitutional, the Supreme Court granted certiorari and consolidated the two cases deciding this one along with Mack v. United States. Question Presented Using the Necessary and Proper Clause of Article I as justification, can Congress temporarily require state CLEOs to regulate handgun purchases by performing those duties called for by the Brady Bill's handgun applicant background-checks? Conclusion No. The Court constructed its opinion on the old principle that state legislatures are not subject to federal direction. The Court explained that while Congress may require the federal government to regulate commerce directly, in this case by performing background-checks on applicants for handgun ownership, the Necessary and Proper Clause does not empower it to compel state CLEOs to fulfill its federal tasks for it - even temporarily. The Court added that the Brady Bill could not require CLEOs to perform the related tasks of disposing of handgun-application forms or notifying certain applicants of the reasons for their refusal in writing, since the Brady Bill reserved such duties only for those CLEO's who voluntarily accepted them

Federal Maritime Commission v. South Carolina Ports Authority

Subjects: Federalism: Public Utilities Facts of the Case South Carolina Maritime Services, Inc. (Maritime Services), asked the South Carolina State Ports Authority (SCSPA) five times for permission to berth a cruise ship, the M/V Tropic Sea, at the SCSPA's port facilities in Charleston, South Carolina. Some cruises offered by Maritime Services would allow passengers to participate in gambling activities while on board. The SCSPA repeatedly denied Maritime Services' requests, contending that it had an established policy of denying berths in the Port of Charleston to vessels whose primary purpose was gambling. Maritime Services file a complaint with the Federal Maritime Commission (FMC), arguing that SCSPA violated the Shipping Act by its denials. The complaint was referred to an Administrative Law Judge (ALJ), who found that the SCSPA, as an arm of the State of South Carolina, was entitled to sovereign immunity and thus dismissed the complaint. Reversing on its own motion, the FMC concluded that state sovereign immunity covers proceedings before judicial tribunals, not Executive Branch agencies. In reversing, Court of Appeals found that the proceedings were an adjudication and thus subject to state sovereign immunity. Question Presented Does a State's sovereign immunity preclude the Federal Maritime Commission from adjudicating a private party's complaint that a state-run port has violated the Shipping Act of 1984? Conclusion Yes. In a 5-4 opinion delivered by Justice Clarence Thomas, the Court held that state sovereign immunity bars the FMC from adjudicating a private party's complaint against a nonconsenting State. Historically, the Court noted, states were not subject to private suits in administrative adjudications when the Constitution was adopted, and states were thus presumptively immune from such actions. Moreover, the Court pointed to the similarities between the FMC's proceedings and civil litigation to conclude that there was no basis for distinguishing between the actions for purposes of sovereign immunity. "Although the Framers likely did not envision the intrusion on state sovereignty at issue in today's case, we are nonetheless confident that it is contrary to their constitutional design," wrote Justice Thomas.

*Buckley v. Valeo (1976)

Subjects: First Amendment: Campaign Spending Facts of the Case In the wake of the Watergate affair, Congress attempted to ferret out corruption in political campaigns by restricting financial contributions to candidates. Among other things, the law set limits on the amount of money an individual could contribute to a single campaign and it required reporting of contributions above a certain threshold amount. The Federal Election Commission was created to enforce the statute. Question Presented Did the limits placed on electoral expenditures by the Federal Election Campaign Act of 1971, and related provisions of the Internal Revenue Code of 1954, violate the First Amendment's freedom of speech and association clauses? Conclusion In this complicated case, the Court arrived at two important conclusions. First, it held that restrictions on individual contributions to political campaigns and candidates did not violate the First Amendment since the limitations of the FECA enhance the "integrity of our system of representative democracy" by guarding against unscrupulous practices. Second, the Court found that governmental restriction of independent expenditures in campaigns, the limitation on expenditures by candidates from their own personal or family resources, and the limitation on total campaign expenditures did violate the First Amendment. Since these practices do not necessarily enhance the potential for corruption that individual contributions to candidates do, the Court found that restricting them did not serve a government interest great enough to warrant a curtailment on free speech and association.

Santa Fe Independent School Dist. v. Doe (2000)

Subjects: First Amendment: Establishment of Religion Facts of the Case Prior to 1995, a student elected as Santa Fe High School's student council chaplain delivered a prayer, described as overtly Christian, over the public address system before each home varsity football game. One Mormon and one Catholic family filed suit challenging this practice and others under the Establishment Clause of the First Amendment. The District Court enjoined the public Santa Fe Independent School District (the District) from implementing its policy as it stood. While the suit was pending, the District adopted a new policy, which permitted, but did not require, student-initiated and student-led prayer at all the home games and which authorized two student elections, the first to determine whether "invocations" should be delivered at games, and the second to select the spokesperson to deliver them. After the students authorized such prayers and selected a spokesperson, the District Court entered an order modifying the policy to permit only nonsectarian, nonproselytizing prayer. The Court of Appeals held that, even as modified by the District Court, the football prayer policy was invalid. The District petitioned for a writ of certiorari, claiming its policy did not violate the Establishment Clause because the football game messages were private student speech, not public speech. Question Presented Does the Santa Fe Independent School District's policy permitting student-led, student-initiated prayer at football games violate the Establishment Clause of the First Amendment? Conclusion Yes. In a 6-3 opinion delivered by Justice John Paul Stevens, the Court held that the District's policy permitting student-led, student-initiated prayer at football games violates the Establishment Clause. The Court concluded that the football game prayers were public speech authorized by a government policy and taking place on government property at government-sponsored school-related events and that the District's policy involved both perceived and actual government endorsement of the delivery of prayer at important school events. Such speech is not properly characterized as "private," wrote Justice Stevens for the majority. In dissent, Chief Justice William H. Rehnquist, joined by Justices Antonin Scalia and Clarence Thomas, noted the "disturbing" tone of the Court's opinion that "bristle[d] with hostility to all things religious in public life."

Engel v. Vitale (1962)

Subjects: First Amendment: Establishment of Religion Facts of the Case The Board of Regents for the State of New York authorized a short, voluntary prayer for recitation at the start of each school day. This was an attempt to defuse the politically potent issue by taking it out of the hands of local communities. The blandest of invocations read as follows: "Almighty God, we acknowledge our dependence upon Thee, and beg Thy blessings upon us, our teachers, and our country." Question Presented Does the reading of a nondenominational prayer at the state of the school day violate the "establishment of religion" clause of the First Amendment? Conclusion Yes. Neither the prayer's nondenominational character nor its voluntary character saves it from unconstitutionality. By providing the prayer, New York officially approved religion. This was the first in a series of cases in which the Court used the establishment clause to eliminate religious activities of all sorts, which had traditionally been a part of public ceremonies. Despite the passage of time, the decision is still unpopular with a majority of Americans.

New York Times v. Sullivan (1964)

Subjects: First Amendment: Libel, Defamation Facts of the Case Decided together with Abernathy v. Sullivan, this case concerns a full-page ad in the New York Times which alleged that the arrest of the Rev. Martin Luther King, Jr. for perjury in Alabama was part of a campaign to destroy King's efforts to integrate public facilities and encourage blacks to vote. L. B. Sullivan, the Montgomery city commissioner, filed a libel action against the newspaper and four black ministers who were listed as endorsers of the ad, claiming that the allegations against the Montgomery police defamed him personally. Under Alabama law, Sullivan did not have to prove that he had been harmed; and a defense claiming that the ad was truthful was unavailable since the ad contained factual errors. Sullivan won a $500,000 judgment. Question Presented Did Alabama's libel law, by not requiring Sullivan to prove that an advertisement personally harmed him and dismissing the same as untruthful due to factual errors, unconstitutionally infringe on the First Amendment's freedom of speech and freedom of press protections? Conclusion The Court held that the First Amendment protects the publication of all statements, even false ones, about the conduct of public officials except when statements are made with actual malice (with knowledge that they are false or in reckless disregard of their truth or falsity). Under this new standard, Sullivan's case collapsed.

Tinker v. Des Moines Ind. Comm. School Dist. (1969)

Subjects: First Amendment: Miscellaneous Facts of the Case John Tinker, 15 years old, his sister Mary Beth Tinker, 13 years old, and Christopher Echardt, 16 years old, decided along with their parents to protest the Vietnam War by wearing black armbands to their Des Moines schools during the christmas holiday season. Upon learning of their intentions, and fearing that the armbands would provoke disturbances, the principals of Des Moins' school districts resolved that all students wearing armbands be asked to remove them or face suspension. When the Tinker siblings and Christopher wore their armbands to school, they were asked to remove them. When they refused, they were suspended until after New Year's Day. Question Presented Does a prohibition against the wearing of armbands in public school, as a form of symbolic protest, violate the First Amendment's freedom of speech protections? Conclusion The wearing of armbands was "closely akin to 'pure speech'" and protected by the First Amendment. School environments imply limitations on free expression, but here the principals lacked justification for imposing any such limits.The principals had failed to show that the forbidden conduct would substantially interfere with appropriate school discipline.

New York Times v. United States (1971)

Subjects: First Amendment: Miscellaneous Facts of the Case In what became known as the "Pentagon Papers Case," the Nixon Administration attempted to prevent the New York Times and Washington Post from publishing materials belonging to a classified Defense Department study regarding the history of United States activities in Vietnam. The President argued that prior restraint was necessary to protect national security. This case was decided together with United States v. Washington Post Co. Question Presented Did the Nixon administration's efforts to prevent the publication of what it termed "classified information" violate the First Amendment? Conclusion Yes. In its per curiam opinion the Court held that the government did not overcome the "heavy presumption against" prior restraint of the press in this case. Justices Black and Douglas argued that the vague word "security" should not be used "to abrogate the fundamental law embodied in the First Amendment." Justice Brennan reasoned that since publication would not cause an inevitable, direct, and immediate event imperiling the safety of American forces, prior restraint was unjustified.

Miller v. California (1973)

Subjects: First Amendment: Obscenity, State Facts of the Case Miller, after conducting a mass mailing campaign to advertise the sale of "adult" material, was convicted of violating a California statute prohibiting the distribution of obscene material. Some unwilling recipients of Miller's brochures complained to the police, initiating the legal proceedings. Question Presented Is the sale and distribution of obscene materials by mail protected under the First Amendment's freedom of speech guarantee? Conclusion In a 5-to-4 decision, the Court held that obscene materials did not enjoy First Amendment protection. The Court modified the test for obscenity established in Roth v. United States and Memoirs v. Massachusetts, holding that "[t]he basic guidelines for the trier of fact must be: (a) whether 'the average person, applying contemporary community standards' would find that the work, taken as a whole, appeals to the prurient interest. . . (b) whether the work depicts or describes, in a patently offensive way, sexual conduct specifically defined by the applicable state law; and (c) whether the work, taken as a whole, lacks serious literary, artistic, political, or scientific value." The Court rejected the "utterly without redeeming social value" test of the Memoirs decision.

*Lemon v. Kurtzman (1971)

Subjects: First Amendment: Parochiaid Facts of the Case This case was heard concurrently with two others, Early v. DiCenso (1971) and Robinson v. DiCenso (1971). The cases involved controversies over laws in Pennsylvania and Rhode Island. In Pennsylvania, a statute provided financial support for teacher salaries, textbooks, and instructional materials for secular subjects to non-public schools. The Rhode Island statute provided direct supplemental salary payments to teachers in non-public elementary schools. Each statute made aid available to "church-related educational institutions." Question Presented Did the Rhode Island and Pennsylvania statutes violate the First Amendment's Establishment Clause by making state financial aid available to "church-related educational institutions"? Conclusion Yes. Writing for the majority, Chief Justice Burger articulated a three-part test for laws dealing with religious establishment. To be constitutional, a statute must have "a secular legislative purpose," it must have principal effects which neither advance nor inhibit religion, and it must not foster "an excessive government entanglement with religion." The Court found that the subsidization of parochial schools furthered a process of religious inculcation, and that the "continuing state surveillance" necessary to enforce the specific provisions of the laws would inevitably entangle the state in religious affairs. The Court also noted the presence of an unhealthy "divisive political potential" concerning legislation which appropriates support to religious schools.

Wallace v. Jaffree (1985)

Subjects: First Amendment: Parochiaid Facts of the Case An Alabama law authorized teachers to conduct regular religious prayer services and activities in school classrooms during the school day. Three of Jaffree's children attended public schools in Mobile. Question Presented Did Alabama law violate the First Amendment's Establishment Clause? Conclusion Yes. The Court determined the constitutionality of Alabama's prayer and meditation statute by applying the secular purpose test, which asked if the state's actual purpose was to endorse or disapprove of religion. The Court held that Alabama's passage of the prayer and meditation statute was not only a deviation from the state's duty to maintain absolute neutrality toward religion, but was an affirmative endorsement of religion. As such, the statute clearly lacked any secular purpose as it sought to establish religion in public schools, thereby violating the First Amendment's Establishment Clause.

Furman v. Georgia (1972)

Three cases were brought to the Supreme Court concerning the death penalty and the racial biases present in the selection process. Three juries had convicted and imposed the death penalty on their accused without any guidelines to go by in their decision. This case represents the first time the Supreme Court ruled against the death penalty. The dissenting Justices argued that the courts had no right to challenge legislative judgement on the effectiveness and justice of punishments. The majority however held that the death penalty was cruel and unusual punishment, which violated the Eigth Amendment. Justice Thurgood Marshall went on to attack the penalty more directly stating, "it is excessive, unneccessary, and offensive to contemporary values." The actual decision created three options for use of the death penalty: mandatory death sentence for certain crimes, development of standardized guidelines for juries, and outright abolition. Later in Gregg v. Georgia the Court favored the creation of guidelines for juries.

Zelman v. Simmons-Harris (2002)

Subjects: First Amendment: Parochiaid Facts of the Case Ohio's Pilot Project Scholarship Program provides tuition aid in the form of vouchers for certain students in the Cleveland City School District to attend participating public or private schools of their parent's choosing. Both religious and nonreligious schools in the district may participate. Tuition aid is distributed to parents according to financial need, and where the aid is spent depends solely upon where parents choose to enroll their children. In the 1999-2000 school year 82 percent of the participating private schools had a religious affiliation and 96 percent of the students participating in the scholarship program were enrolled in religiously affiliated schools. Sixty percent of the students were from families at or below the poverty line. A group of Ohio taxpayers sought to enjoin the program on the ground that it violated the Establishment Clause. The District Court granted them summary judgment, and the Court of Appeals affirmed. Question Presented Does Ohio's school voucher program violate the Establishment Clause? Conclusion No. In a 5-4 opinion delivered by Chief Justice William H. Rehnquist, the Court held that the program does not violate the Establishment Clause. The Court reasoned that, because Ohio's program is part of Ohio's general undertaking to provide educational opportunities to children, government aid reaches religious institutions only by way of the deliberate choices of numerous individual recipients and the incidental advancement of a religious mission, or any perceived endorsement, is reasonably attributable to the individual aid recipients not the government. Chief Justice Rehnquist wrote that the "Ohio program is entirely neutral with respect to religion. It provides benefits directly to a wide spectrum of individuals, defined only by financial need and residence in a particular school district. It permits such individuals to exercise genuine choice among options public and private, secular and religious. The program is therefore a program of true private choice."

BOE of Westside Community Schools v. Mergens (1990)

Subjects: First Amendment: Parochiaid Facts of the Case The school administration at Westside High School denied permission to a group of students to form a Christian club with the same privileges and meeting terms as other Westside after-school student clubs. In addition to citing the Establishment Clause, Westside refused the club's formation because it lacked a faculty sponsor. When the school board upheld the administration's denial, Mergens and several other students sued. The students alleged that Westside's refusal violated the Equal Access Act, which requiremes that schools in receipt of federal funds provide "equal access" to student groups seeking to express "religious, political, philosophical, or other content" messages. On appeal from an adverse District Court ruling, the Court of Appeals found in favor of the students. The Supreme Court granted Westside certiorari. Question Presented Was Westside's prohibition against the formation of a Christian club consistent with the Establishment Clause, thereby rendering the Equal Access Act unconstitutional? Conclusion No. In distinguishing between "curriculum" and "noncurriculum student groups," the Court held that since Westside permitted other noncurricular clubs, it was prohibited under the Equal Access Act from denying equal access to any after-school club based on the content of its speech. The proposed Christian club would be a noncurriculum group since no other course required students to become its members, its subject matter would not actually be taught in classes, it did not concern the school's cumulative body of courses, and its members would not receive academic credit for their participation. The Court added that the Equal Access Act was constitutional because it served an overriding secular purpose by prohibiting discrimination on the basis of philosophical, political, or other types of speech. As such, the Act protected the Christian club's formation even if its members engaged in religious discussions.

Bush v. Gore (2000)

Subjects: Judicial Power: Civil Rights Facts of the Case Following the U.S. Supreme Court's decision in Bush v. Palm Beach County Canvassing Board, and concurrent with Vice President Al Gore's contest of the certification of Florida presidential election results, on December 8, 2000 the Florida Supreme Court ordered that the Circuit Court in Leon County tabulate by hand 9000 contested ballots from Miami-Dade County. It also ordered that every county in Florida must immediately begin manually recounting all "under-votes" (ballots which did not indicate a vote for president) because there were enough contested ballots to place the outcome of the election in doubt. Governor George Bush and his running mate, Richard Cheney, filed a request for review in the U.S. Supreme Court and sought an emergency petition for a stay of the Florida Supreme Court's decision. The U.S. Supreme Court granted review and issued the stay on December 9. It heard oral argument two days later. Question Presented Did the Florida Supreme Court violate Article II Section 1 Clause 2 of the U.S. Constitution by making new election law? Do standardless manual recounts violate the Equal Protection and Due Process Clauses of the Constitution? Conclusion Noting that the Equal Protection clause guarantees individuals that their ballots cannot be devalued by "later arbitrary and disparate treatment," the per curiam opinion held 7-2 that the Florida Supreme Court's scheme for recounting ballots was unconstitutional. Even if the recount was fair in theory, it was unfair in practice. The record suggested that different standards were applied from ballot to ballot, precinct to precinct, and county to county. Because of those and other procedural difficulties, the court held that no constitutional recount could be fashioned in the time remaining (which was short because the Florida legislature wanted to take advantage of the "safe harbor" provided by 3 USC Section 5). Loathe to make broad precedents, the per curiam opinion limited its holding to the present case. Rehnquist (in a concurring opinion joined by Scalia and Thomas) argued that the recount scheme was also unconstitutional because the Florida Supreme Court's decision made new election law, which only the state legislature may do. Breyer and Souter (writing separately) agreed with the per curiam holding that the Florida Court's recount scheme violated the Equal Protection Clause, but they dissented with respect to the remedy, believing that a constitutional recount could be fashioned. Time is insubstantial when constitutional rights are at stake. Ginsburg and Stevens (writing separately) argued that for reasons of federalism, the Florida Supreme Court's decision ought to be respected. Moreover, the Florida decision was fundamentally right; the Constitution requires that every vote be counted.

Wesberry v. Sanders (1964)

Subjects: Judicial Power: Standing to Sue, Justiciable Question Facts of the Case James P. Wesberry, Jr. filed a suit against the Governor of Georgia, Carl E. Sanders, protesting the state's apportionment scheme. The Fifth Congressional District, of which Wesberry was a member, had a population two to three times larger than some of the other districts in the state. Wesberry claimed this system diluted his right to vote compared to other Georgia residents. Question Presented Did Georgia's congressional districts violate the Fourteenth Amendment or deprive citizens of the full benefit of their right to vote? Conclusion The Court held that Georgia's apportionment scheme grossly discriminated against voters in the Fifth Congressional District. Because a single congressman had to represent two to three times as many people as were represented by congressmen in other districts, the Georgia statute contracted the value of some votes and expanded the value of others. The Court recognized that "no right is more precious" than that of having a voice in elections and held that "[t]o say that a vote is worth more in one district than in another would not only run counter to our fundamental ideas of democratic government, it would cast aside the principle of a House of Representatives elected 'by the People. . .'"

Powell v. McCormack (1969)

Subjects: Judicial Power: Standing to Sue, Justiciable Question Facts of the Case Adam Clayton Powell pecked at his fellow representatives from his unassailable perch in New York's Harlem. Powell had been embroiled in controversy inside and outside Washington. When Powell failed to heed civil proceedings against him in New York, a judge held him in criminal contempt. His problems were only beginning. He won reelection in 1966 but the House of Representatives voted to exclude him. Question Presented May the House of Representatives exclude a duly elected member if the member has satisfied the standing requirements of age, citizenship and residence as articulated in Article I Section 2 of the U.S. Constitution? Conclusion No. The Court noted that the proceedings against Powell were intended to exclude and not expel him from the chamber. That is an important distinction to recognize since the House does have the power under Article I, Section 5 to expel members. However, expulsion was not the purpose of the proceedings in this case. After analyzing the Framers' debates on this issue, Chief Justice Warren concluded that since Powell had been lawfully elected by his constituents and since he met the constitutional requirements for membership in the House, that the chamber was powerless to exclude him

Reynolds v. Sims (1964)

Subjects: Judicial Power: Standing to Sue, Justiciable Question Facts of the Case In 1961, M.O. Sims, David J. Vann (Vann v. Baggett), John McConnell (McConnell v. Baggett), and other voters from Jefferson County, Alabama, challenged the apportionment of the state legislature. The Alabama Constitution prescribed that each county was entitled to at least one representative and that there were to be as many senatorial districts as there were senators. Population variance ratios of as great as 41-to-1 existed in the Senate. Question Presented Did Alabama's apportionment scheme violate the Fourteenth Amendment's Equal Protection Clause by mandating at least one representative per county and creating as many senatorial districts as there were senators, reguardless of population variances? Conclusion In an 8-to-1 decision, the Court upheld the challenge to the Alabama system, holding that Equal Protection Clause demanded "no less than substantially equal state legislative representation for all citizens...." Noting that the right to direct representation was "a bedrock of our political system," the Court held that both houses of bicameral state legislatures had to be apportioned on a population basis. States were required to "honest and good faith" efforts to construct districts as nearly of equal population as practicable.

*Baker v. Carr (1962)

Subjects: Judicial Power: Standing to Sue, Legal Injury Facts of the Case Charles W. Baker and other Tennessee citizens alleged that a 1901 law designed to apportion the seats for the state's General Assembly was virtually ignored. Baker's suit detailed how Tennessee's reapportionment efforts ignored significant economic growth and population shifts within the state. Question Presented Did the Supreme Court have jurisdiction over questions of legislative apportionment? Conclusion In an opinion which explored the nature of "political questions" and the appropriateness of Court action in them, the Court held that there were no such questions to be answered in this case and that legislative apportionment was a justiciable issue. In his opinion, Justice Brennan provided past examples in which the Court had intervened to correct constitutional violations in matters pertaining to state administration and the officers through whom state affairs are conducted. Brennan concluded that the Fourteenth Amendment equal protection issues which Baker and others raised in this case merited judicial evaluation.

*Griswold v. Connecticut (1965)

Subjects: Judicial Power: Standing to Sue, Personal Injury Facts of the Case Griswold was the Executive Director of the Planned Parenthood League of Connecticut. Both she and the Medical Director for the League gave information, instruction, and other medical advice to married couples concerning birth control. Griswold and her colleague were convicted under a Connecticut law which criminalized the provision of counselling, and other medical treatment, to married persons for purposes of preventing conception. Question Presented Does the Constitution protect the right of marital privacy against state restrictions on a couple's ability to be counseled in the use of contraceptives? Conclusion Though the Constitution does not explicitly protect a general right to privacy, the various guarantees within the Bill of Rights create penumbras, or zones, that establish a right to privacy. Together, the First, Third, Fourth, and Ninth Amendments, create a new constitutional right, the right to privacy in marital relations. The Connecticut statute conflicts with the exercise of this right and is therefore null and void.

INS (Immigration and naturalization Service) v. Chadha (1983)

Subjects: Miscellaneous: Legislative Veto Facts of the Case In one section of the Immigration and Nationality Act, Congress authorized either House of Congress to invalidate and suspend deportation rulings of the United States Attorney General. Chadha had stayed in the U.S. past his visa deadline and was ordered to leave the country. The House of Representatives suspended the Immigration judge's deportation ruling. This case was decided together with United States House of Representatives v. Chadha and United States Senate v. Chadha. Question Presented Did the Immigration and Nationality Act, which allowed a one-House veto of executive actions, violate the separation of powers doctrine? Conclusion The Court held that the particular section of the Act in question did violate the Constitution. Recounting the debates of the Constitutional Convention over issues of bicameralism and separation of powers, Chief Justice Burger concluded that even though the Act would have enhanced governmental efficiency, it violated the "explicit constitutional standards" regarding lawmaking and congressional authority.

*Clinton v. City of New York (1998)

Subjects: Miscellaneous: Miscellaneous Facts of the Case This case consolidates two separate challenges to the constitutionality of two cancellations, made by President William J. Clinton, under the Line Item Veto Act ("Act"). In the first, the City of New York, two hospital associations, a hospital, and two health care unions, challenged the President's cancellation of a provision in the Balanced Budget Act of 1997 which relinquished the Federal Government's ability to recoup nearly $2.6 billion in taxes levied against Medicaid providers by the State of New York. In the second, the Snake River farmer's cooperative and one of its individual members challenged the President's cancellation of a provision of the Taxpayer Relief Act of 1997. The provision permitted some food refiners and processors to defer recognition of their capital gains in exchange for selling their stock to eligible farmers' cooperatives. After a district court held the Act unconstitutional, the Supreme Court granted certiorari on expedited appeal. Question Presented Did the President's ability to selectively cancel individual portions of bills, under the Line Item Veto Act, violate the Presentment Clause of Article I? Conclusion Yes. In a 6-to-3 decision the Court first established that both the City of New York, and its affiliates, and the farmers' cooperative suffered sufficiently immediate and concrete injuries to sustain their standing to challenge the President's actions. The Court then explained that under the Presentment Clause, legislation that passes both Houses of Congress must either be entirely approved (i.e. signed) or rejected (i.e. vetoed) by the President. The Court held that by canceling only selected portions of the bills at issue, under authority granted him by the Act, the President in effect "amended" the laws before him. Such discretion, the Court concluded, violated the "finely wrought" legislative procedures of Article I as envisioned by the Framers.

Webster v. Reproductive Health Services (1989)

Subjects: Privacy: Abortion, Including Contraceptive Facts of the Case In 1986, the state of Missouri enacted legislation that placed a number of restrictions on abortions. The statute's preamble indicated that "[t]he life of each human being begins at conception," and the law codified the following restrictions: public employees and public facilities were not to be used in performing or assisting abortions unnecessary to save the mother's life; encouragement and counseling to have abortions was prohibited; and physicians were to perform viability tests upon women in their twentieth (or more) week of pregnancy. Lower courts struck down the restrictions. Question Presented Did the Missouri restrictions unconstitutionally infringe upon the right to privacy or the Equal Protection Clause of the Fourteenth Amendment? Conclusion In a controversial and highly fractured decision, the Court held that none of the challenged provisions of the Missouri legislation were unconstitutional. First, the Court held that the preamble had not been applied in any concrete manner for the purposes of restricting abortions, and thus did not present a constitutional question. Second, the Court held that the Due Process Clause did not require states to enter into the business of abortion, and did not create an affirmative right to governmental aid in the pursuit of constitutional rights. Third, the Court found that no case or controversy existed in relation to the counseling provisions of the law. Finally, the Court upheld the viability testing requirements, arguing that the State's interest in protecting potential life could come into existence before the point of viability. The Court emphasized that it was not revisiting the essential portions of the holding in Roe v. Wade.

* Roe v. Wade (1973)

Subjects: Privacy: Abortion, Including Contraceptives Facts of the Case Roe, a Texas resident, sought to terminate her pregancy by abortion. Texas law prohibited abortions except to save the pregnant woman's life. After granting certiorari, the Court heard arguments twice. The first time, Roe's attorney -- Sarah Weddington -- could not locate the constitutional hook of her argument for Justice Potter Stewart. Her opponent -- Jay Floyd -- misfired from the start. Weddington sharpened her constitutional argument in the second round. Her new opponent -- Robert Flowers -- came under strong questioning from Justices Potter Stewart and Thurgood Marshall. Question Presented Does the Constitution embrace a woman's right to terminate her pregnancy by abortion? Conclusion The Court held that a woman's right to an abortion fell within the right to privacy (recognized in Griswold v. Connecticut) protected by the Fourteenth Amendment. The decision gave a woman total autonomy over the pregnancy during the first trimester and defined different levels of state interest for the second and third trimesters. As a result, the laws of 46 states were affected by the Court's ruling.

Planned Parenthood v. Casey (1992)

Subjects: Privacy: Abortion, Including Contraceptives Facts of the Case The Pennsylvania legislature amended its abortion control law in 1988 and 1989. Among the new provisions, the law required informed consent and a 24 hour waiting period prior to the procedure. A minor seeking an abortion required the consent of one parent (the law allows for a judicial bypass procedure). A married woman seeking an abortion had to indicate that she notified her husband of her intention to abort the fetus. These provisions were challenged by several abortion clinics and physicians. A federal appeals court upheld all the provisions except for the husband notification requirement. Question Presented Can a state require women who want an abortion to obtain informed consent, wait 24 hours, and, if minors, obtain parental consent, without violating their right to abortions as guaranteed by Roe v. Wade? Conclusion In a bitter, 5-to-4 decision, the Court again reaffirmed Roe, but it upheld most of the Pennsylvania provisions. For the first time, the justices imposed a new standard to determine the validity of laws restricting abortions. The new standard asks whether a state abortion regulation has the purpose or effect of imposing an "undue burden," which is defined as a "substantial obstacle in the path of a woman seeking an abortion before the fetus attains viability." Under this standard, the only provision to fail the undue-burden test was the husband notification requirement. The opinion for the Court was unique: It was crafted and authored by three justices.

*United States v. Morrison (2000)

Subjects:Civil Rights: Liability, Civil Rights Acts Facts of the Case In 1994, while enrolled at Virginia Polytechnic Institute (Virginia Tech), Christy Brzonkala alleged that Antonio Morrison and James Crawford, both students and varsity football players at Virginia Tech, raped her. In 1995, Brzonkala filed a complaint against Morrison and Crawford under Virginia Tech's Sexual Assault Policy. After a hearing, Morrison was found guilty of sexual assault and sentenced to immediate suspension for two semesters. Crawford was not punished. A second hearing again found Morrison guilty. After an appeal through the university's administrative system, Morrison's punishment was set aside, as it was found to be "excessive." Ultimately, Brzonkala dropped out of the university. Brzonkala then sued Morrison, Crawford, and Virginia Tech in Federal District Court, alleging that Morrison's and Crawford's attack violated 42 USC section 13981, part of the Violence Against Women Act of 1994 (VAWA), which provides a federal civil remedy for the victims of gender-motivated violence. Morrison and Crawford moved to dismiss Brzonkala's suit on the ground that section 13981's civil remedy was unconstitutional. In dismissing the complaint, the District Court found that that Congress lacked authority to enact section 13981 under either the Commerce Clause or the Fourteenth Amendment, which Congress had explicitly identified as the sources of federal authority for it. Ultimately, the Court of Appeals affirmed. Question Presented Does Congress have the authority to enact the Violence Against Women Act of 1994 under either the Commerce Clause or Fourteenth Amendment? Conclusion No. In a 5-4 opinion delivered by Chief Justice William H. Rehnquist, the Court held that Congress lacked the authority to enact a statute under the Commerce Clause or the Fourteenth Amendment since the statute did not regulate an activity that substantially affected interstate commerce nor did it redress harm caused by the state. Chief Justice Rehnquist wrote for the Court that [i]f the allegations here are true, no civilized system of justice could fail to provide [Brzonkala] a remedy for the conduct of...Morrison. But under our federal system that remedy must be provided by the Commonwealth of Virginia, and not by the United States." Dissenting, Justice Stephen G. Breyer argued that the majority opinion "illustrates the difficulty of finding a workable judicial Commerce Clause touchstone." Additionally, Justice David H. Souter, dissenting, noted that VAWA contained a "mountain of data assembled by Congress...showing the effects of violence against women on interstate commerce."

South Dakota v. Dole (1987)

Subjects:Federalism: Miscellaneous Facts of the Case In 1984, Congress enacted legislation ordering the Secretary of Transportation to withhold five percent of federal highway funds from states that did not adopt a 21-year-old minimum drinking age. South Dakota, a state that permitted persons 19 years of age to purchase alcohol, challenged the law. Question Presented Did Congress exceed its spending powers, or violate the Twenty-first Amendment, by passing legislation conditioning the award of federal highway funds on the states' adoption of a uniform minimum drinking age? Conclusion No. In a 7-to-2 decision, the Court held that Congress, acting indirectly to encourage uniformity in states' drinking ages, was within constitutional bounds. The Court found that the legislation was in pursuit of "the general welfare," and that the means chosen to do so were reasonable. The Court also held that the Twenty-first Amendment's limitations on spending power were not prohibitions on congressional attempts to achieve federal objectives indirectly. The five percent loss of highway funds was not unduly coercive.

Sheppard v. Maxwell 1966

The media coverage of Dr. Sam Sheppard's arrest for the murder of his wife was extensive. The brutal nature of the crime, the relationship between the victim and the accused, and the social position of the accused all helped to fuel the public's interest in the trial. Sheppard argued that this media coverage prevented him from receiving a fair trial. The Supreme Court agreed with him and granted the writ of habeas corpus he had requested. The reversal of the decision was justified by the Court through the "Roman Holiday" atmosphere of the trial in which the judge "failed to minimize the prejudicial impact of massive publicity." The Sheppard decision allowed the use of the "gag" order to limit pretrial publicity

California v. Greenwood (6-2 vote, May 16, 1988)

The police, acting on a tip that Billy Greenwood was dealing in illegal narcotics, searched trash bags out on his curb. Finding paraphernalia associated with drug use in the garbage bags, the police applied for a search warrant; they included in their affidavits descriptions of what they had found in the garbage. Based on evidence from both the garbage can and the subsequent search, Greenwood was convicted of drug-related charges. He appealed to the Supreme Court, claiming that the search of his garbage was warrantless and therefore illegal. The Supreme Court upheld the search, claiming that the bags of trash left on the side of the street were open to inspection by "animals, children, scavengers, snoops, and other members of the public." Justices Marshall and Brennan dissented, reasoning the search of another person's garbage did not constitute civilized behavior, and therefore a person has the right to expect that the contents of any opaque container thrown out or transported will remain private. Nevertheless, the garbage was ruled public domain and the evidence was admissable.

Weeks v. United States (9-0 vote, February 24, 1914)

Weeks illustrates the Supreme Court's first use of the "exclusionary rule." The case concerns a man who was subject to a warrantless search by the police and deputy marshal. While searching Week's premises, the law enforcement agents discovered lottery tickets in his mail. Based on this evidence, Weeks was prosecuted for the illegal transport of gambling items. Prior to the trial, he requested the items taken from his house be returned; during the trial, he objected to their being offered into evidence. This provided the grounds for the Supreme Court appeal. The Court found in favor of Weeks, stating in their decision "...if letters and private documents can thus be seized and used as evidence...his right to be secure against such searches... is of no value, and...might as well be stricken from the Constitution."

Dennis v. United States 1951

his case concerns the Smith Act, which was enacted in order to prevent the spread of Communism in the United States. This law made it a crime to plot the violent overthrow of government, even if no physical steps were taken towards that end. It was not employed until the late 1940's due to the war time alliance between the USSR and the USA. After the war, relations between the two countries quickly broke down, bringing about the feeling of an impending domestic Communist threat. Eleven Communist leaders were arrested and charged with planning the violent overthrow of our government, strictly prohibited by the Smith Act. The Communists were convicted, but they then appealed, contending the Smith Act was unconstitutional. The Supreme Court ruled in favor of the government, stating in its decision "In each case, the courts must ask whether the gravity of the 'evil' discounted by its improbability, justifies such invasion of free speech as is necessary to avoid danger."


Kaugnay na mga set ng pag-aaral

Ch. 9 section 4.3, 4.4, 4.5 Virtual Network Devices

View Set

Anatomy Final Lecture New Material

View Set

Fundamentals Chapter 28 Wounds (Prep U)

View Set

Urinary System Exam Review (Pt. 1)

View Set